Chain Please Don't Fall!

A uniform chain of length "L" and mass per unit length is " λ \lambda " is suspended at one end A by inextensible light string and the other end of the chain B is held at rest at the level of end A of the chain (As Shown ). Now The end B of the chain is released under gravity then find the tension in the string at the moment when end B as fallen by distance " y " .


If Tension is expressed as :

T ( y ) = λ g a ( L + b y ) T(y)\quad =\quad \cfrac { \lambda g }{ a } (L\quad +\quad by) .

Then Evaluate the value of "a + b" ?


\bullet Where "a" and "b" are positive co-prime integers .

\bullet Figure is not drawn to scale .

\bullet Variable mass concept Might Be helpful here :)

This is part of my set Deepanshu's Mechanics Blasts


The answer is 5.

This section requires Javascript.
You are seeing this because something didn't load right. We suggest you, (a) try refreshing the page, (b) enabling javascript if it is disabled on your browser and, finally, (c) loading the non-javascript version of this page . We're sorry about the hassle.

2 solutions

Deepanshu Gupta
Nov 18, 2014

Let Left part of chain as system at time t=t. mass of left part is :

m = ( L 2 + y 2 ) λ d m d t = λ 2 d y d t = λ 2 v = λ 2 2 g y m\quad =\quad (\cfrac { L }{ 2 } +\cfrac { y }{ 2 } )\lambda \quad \quad \quad \\ \\ \cfrac { dm }{ dt } \quad =\quad \cfrac { \lambda }{ 2 } \cfrac { dy }{ dt } \quad \\ \quad \quad \quad \quad \quad \\ \quad \quad \quad \quad \quad =\quad \cfrac { \lambda }{ 2 } v\quad \\ \quad \quad \quad \quad \quad =\quad \cfrac { \lambda }{ 2 } \sqrt { 2gy } .


By writing dynamics equation of left chain system :

F e x t F t h r u s t = m a ( a s p o s i t i v e ) a = 0 F t h r u s t = U r e l a t i v e d m d t = λ g y . . . . ( 1 ) F e x t = T m g = T ( L 2 + y 2 ) λ g . . . . ( 2 ) T = λ g 2 ( L + 3 y ) { F }_{ ext }\quad \quad -\quad { F }_{ thrust }\quad =\quad ma\quad \quad (\quad \uparrow \quad as\quad positive\quad )\\ \\ \because \quad a\quad =\quad 0\quad \\ \\ \because \quad { F }_{ thrust }\quad =\quad { U }_{ relative }\left| \cfrac { dm }{ dt } \right| \\ \quad \quad \quad \quad \quad \quad \quad =\quad \lambda gy\quad \quad \quad \quad .\quad .\quad .\quad .\quad (1)\\ \\ \quad { F }_{ ext }\quad =\quad T\quad -\quad mg\\ \quad \quad \quad \quad \quad \\ \quad \quad \quad \quad =\quad T\quad -\quad (\cfrac { L }{ 2 } +\cfrac { y }{ 2 } )\lambda g\quad .\quad .\quad .\quad .\quad (2)\\ \\ \boxed { T\quad =\quad \cfrac { \lambda g }{ 2 } (L\quad +\quad 3y\quad ) } .

Hi Deepanshu. Although I solved it but I am not sure if my method is correct or not. Here is my solution---

If the top of the right half of the chain is lowered by distance y y then the length of the left half of the chain will increase by y / 2 y/2 . If that would have been at rest then the tension in the string will be λ g ( L + y ) 2 \lambda \frac { g(L+y) }{ 2 } . But there will be additional tension in the string due to the the change in the momentum of the last link. Now d p = 2 g x λ d x dp=\sqrt { 2gx } \lambda dx ( Let the last link fall x x distance)

so d p d t = 2 g x λ d x d t \frac { dp }{ dt } =\sqrt { 2gx } \lambda \frac { dx }{ dt }

or d p d t = λ 2 g x \frac { dp }{ dt } =\lambda 2gx

Now x = y 2 x=\frac{y}{2}

Substituting this we will get

d p d t = λ g y \frac { dp }{ dt } =\lambda gy

So tension in the string will be g λ ( L + y ) 2 + λ g y g\lambda \frac { (L+y) }{ 2 }+\lambda gy

So T = λ g 2 ( L + 3 y ) T=\frac { \lambda g }{ 2 } (L+3y)

satvik pandey - 6 years, 6 months ago

Log in to reply

Nice one !

Deepanshu Gupta - 6 years, 6 months ago

Log in to reply

Thanks! :D

satvik pandey - 6 years, 6 months ago

it's absolutely correct.

A Former Brilliant Member - 4 years, 9 months ago

Can you explain me your solution?

Ninad Akolekar - 6 years, 6 months ago

Nice one .

jinay patel - 6 years, 4 months ago

Lovely sum and solution.

rajdeep brahma - 3 years, 2 months ago

Sorry to say that i think your answer is absolutely wrong... The velocity of end of left part of chain is not (2gy)^1/2. We should apply energy conservation to find the velocity. For more information please see this problem- https://brilliant.org/problems/faster-than-gravity/

Rahul Kumar - 4 years, 8 months ago

Log in to reply

After applying energy conservation the velocity for the right part comes out to be a function ........ V=√[gy/2(1-y/L)] sohow could u apply simply it as √ 2gy

Purnima Saxena - 4 years, 2 months ago

Log in to reply

That's what i am asking!

Rahul Kumar - 4 years, 1 month ago

Energy is not conserved as there are inelastic collisions between the links of
the chain

saptarshi dasgupta - 3 years, 2 months ago

@Deepanshu Gupta - Please reply

Rahul Kumar - 4 years, 8 months ago

Let V be the velocity of the whole chain center of mass and v - the point B velocity.

Then the whole chain impulse M V MV equals the impulse of the moving part of the chain :

M / L ( L / 2 y / 2 ) v i . e . , M V = M / L ( L / 2 y / 2 ) v M/L (L/2 - y/2)v i.e., MV=M/L(L/2 - y/2)v .

Differentiating both parts of this equation one gets (with the use of Newton's second law)

M g T = M g / 2 M g y / 2 L M v 2 / 2 L . A s v 2 = 2 g y , T = M g / 2 + 3 M g / 2 L . H e n c e a + b = 5 Mg - T = Mg/2 - Mgy/2L - Mv^2/2L. As v^2=2gy, T = Mg/2 + 3Mg/2L. Hence a + b = 5 .

0 pending reports

×

Problem Loading...

Note Loading...

Set Loading...